12
$\begingroup$

Background:

It is known that every Banach space $X$ can be embedded isometrically as a subspace in the space $C(K)$ of continuous functions on a compact Hausdorff space $K$. Indeed, one can take $K$ equal to the closed unit ball in the dual $X^*$, and embed $X$ in $C((X^*)_1)$ using the duality mapping $x \mapsto \hat{x}\big|_{(X^*)_1} \in C((X^*)_1)$, where $\hat{x} \in X^{**}$ is given by $\hat{x}(f) = f(x)$, for $f \in X^*$. By the Gelfand-Naimark theorem we know that $C(K)$ can be faithfully represented as a sub-C*-algebra of the algebra $B(H)$ of bounded linear operators on some Hilbert space $H$, and so we conclude:

Every Banach space can be represented isometrically as a closed subspace of $B(H)$ for some Hilbert space $H$.

Question:

Which (finite dimensional) Banach spaces can be represented isometrically as closed subspaces of $B(H)$ for a finite dimensional Hilbert space $H$?

A related questions is: do you know any work related to this question?

I usually have spaces of the complex numbers in mind, but I'll also be happy to hear about real spaces.

Why did I ask myself this question?

Nothing fancy. I was giving a lecture about operator systems to undergraduates and I started from operator spaces, but since they didn't all take a course in functional analysis I worked mostly in $M_n$. I made a point that every banach space is an operator space in $B(H)$ for some $H$, and showed them how $\ell^2_2$ and $\ell^\infty_2$ (the space $\mathbb{C}^2$ with the $\ell^2$ and $\ell^\infty$ norms, respectively) can be isometrically embedded in $M_2$. My older notes said that $\ell^1_2$ cannot be embedded in $M_n$ for finite $n$ (I forgot the complete details of the proof but I'll write the idea below). Speaking about this I got curious about which finite dimensional spaces can be embedded in $M_n$ for finite $n$.

The simplest question is:

Can $\ell^1_2$ be embedded in $M_n$?

For this simple question I think that the answer is: no. The reason I think this is that $\ell^1_2$ has a unique operator space structure. We have an isometric representation as the operator subspace of the C-algebra $C(S^1)$ generated by the unitaries $1$ and $z$. Since unitaries are hyperrigid, any unital isometric map from $span\{1,z\}$ onto a subspace of $M_n$ would extend to $*$-isomorphism between the generated C-algebras, and this is impossible. Thus, $\ell^1_2$ is not a unital operator space. I haven't worked much in the nonunital setting, I think that in this particular case the technical difference between unital and nonunital won't change the end result. But then I got stuck (Gupta and Reza's linked paper below gives an elementary proof that $\ell^1_1$ cannot be embedded in $M_n$, and one of the steps is a nice trick that shows how indeed you can reduce to the case of unital operator space).

Update:

Bunyamin Sari, in a comment to Bill Johnson's answer, put a reference to a paper by Samya Kumar Ray showing that $\ell^p_n$ cannot be embedded isometrically as compact operators. That papers contains a reference to a paper by Gupta and Reza that provides an elementary proof that $\ell^1_2$ cannot be embedded in $M_n$. It follows, of course, that no $\ell^1_m$ can be embedded in $M_n$.

Terry Tao commented below: "In order to embed for a finite dimensional Banach space to embed isometrically into some $M_n$ it is necessary that the unit ball be a semi-algebraic set."

To see this, note that the unit ball of an operator space is the intersection of a linear subspace with the unit ball of $M_n$, which can be given as the set of matrices satisfying the matrix inequality $A^*A \leq I$. However, having a semi-algebraic unit ball is not a sufficient condition for embeddability as an operator subspace of $M_n$; for example consider $\ell^p_2$ for $p=4$ and see the paper by Ray above.

(Thanks to Guy Salomon and Eli Shamovich for some offline discussions).

$\endgroup$
5
  • 2
    $\begingroup$ In other words: equip $M_n$ with operator norm and characterize its linear subspaces as normed spaces. I'm not sure there is much we can say about this; it's certainly a very special property. $\endgroup$
    – Nik Weaver
    Jan 9, 2022 at 13:47
  • 3
    $\begingroup$ Any Banach space where the unit ball is a polyhedron is ok just from spaces of diagonal operators. $\endgroup$
    – Will Sawin
    Jan 9, 2022 at 14:50
  • 1
    $\begingroup$ I have never seen this questioned addressed. I guess the starting point is to give examples of finite dimensional spaces that do not embed isometrically into $M_n$ for any $n$. Do you know of some? $\endgroup$ Jan 9, 2022 at 18:47
  • $\begingroup$ Thanks. I think that $\ell^1_2$ (that is, $\mathbb{C}^2$ with the $\ell^1$ norm) can't be embedded in $M_n$ for any $n$. I think that I used to know how to prove it but am missing some details (morally speaking because $\ell^1_2$ has a unique operator space structure; and we have a representation of this space as the subspace of $C(S^1)$ generated by the unitaries $1$ and $z$, so $\ell^1_2$ cannot be represented as a unital operator space. This is because unitaries are "hyperrigid"... this getting too long so I'll add stuff to the question. $\endgroup$
    – Orr Shalit
    Jan 9, 2022 at 19:15
  • 4
    $\begingroup$ In order to embed for a finite dimensional Banach space to embed isometrically into some $M_n$ it is necessary that the unit ball be a semi-algebraic set. $\endgroup$
    – Terry Tao
    Jan 9, 2022 at 21:01

1 Answer 1

6
$\begingroup$

In Ray - On Isometric Embedding $\ell_p^m \to S_\infty^n$ and Unique operator space structure it is shown that for $p\in (2,\infty)\cup \{1\}$ and $n\ge 2$, $\ell_p^n$ does not isometrically embed into the space of compact operators on $\ell_2$.

(This was previously in the comments, now deleted for housekeeping.)

$\endgroup$
1
  • $\begingroup$ Thanks. The fact that there are several recent papers on this problem, together with the fact that having semi-algebraic unit ball is necessary but not sufficient, gives me the feeling that this is going to be the best answer. But I'll wait a few days before accepting it. $\endgroup$
    – Orr Shalit
    Jan 11, 2022 at 5:30

Your Answer

By clicking “Post Your Answer”, you agree to our terms of service and acknowledge you have read our privacy policy.

Not the answer you're looking for? Browse other questions tagged or ask your own question.